Difference between revisions of "1957 AHSME Problems/Problem 2"

(Solution)
m (typo fix)
 
(One intermediate revision by the same user not shown)
Line 18: Line 18:
 
\end{align*}
 
\end{align*}
  
Thus, our answer is <math>\boxed{\textbf{(E) } 8 \text{ and } 3}</math>.
+
Thus, our answer is <math>\boxed{\textbf{(E) } 8 \text{ and } -3}</math>.
  
 
== See also ==
 
== See also ==
  
{{AHSME 50p box|year=1957|before=First Problem|num-a=2}}
+
{{AHSME 50p box|year=1957|num-b=1|num-a=3}}
 
{{MAA Notice}}
 
{{MAA Notice}}
 
[[Category:AHSME]][[Category:AHSME Problems]]
 
[[Category:AHSME]][[Category:AHSME Problems]]
 
[[Category:Introductory Algebra Problems]]
 
[[Category:Introductory Algebra Problems]]

Latest revision as of 11:06, 24 July 2024

Problem

In the equation $2x^2 - hx + 2k = 0$, the sum of the roots is $4$ and the product of the roots is $-3$. Then $h$ and $k$ have the values, respectively:

$\textbf{(A)}\ 8\text{ and }{-6} \qquad  \textbf{(B)}\ 4\text{ and }{-3}\qquad  \textbf{(C)}\ {-3}\text{ and }4\qquad \textbf{(D)}\ {-3}\text{ and }8\qquad \textbf{(E)}\ 8\text{ and }{-3}$

Solution

Let the roots of the given equation be $r$ and $s$. Then, by Vieta's Formulas, we have the following: \begin{align*} \frac{h}{2} = r+s = 4 &\rightarrow h = 8 \\ \frac{2k}{2} = rs = -3 &\rightarrow k = -3 \\ \end{align*}

Thus, our answer is $\boxed{\textbf{(E) } 8 \text{ and } -3}$.

See also

1957 AHSC (ProblemsAnswer KeyResources)
Preceded by
Problem 1
Followed by
Problem 3
1 2 3 4 5 6 7 8 9 10 11 12 13 14 15 16 17 18 19 20 21 22 23 24 25 26 27 28 29 30 31 32 33 34 35 36 37 38 39 40 41 42 43 44 45 46 47 48 49 50
All AHSME Problems and Solutions

The problems on this page are copyrighted by the Mathematical Association of America's American Mathematics Competitions. AMC logo.png